博弈论答疑

  1. 1、下载文档前请自行甄别文档内容的完整性,平台不提供额外的编辑、内容补充、找答案等附加服务。
  2. 2、"仅部分预览"的文档,不可在线预览部分如存在完整性等问题,可反馈申请退款(可完整预览的文档不适用该条件!)。
  3. 3、如文档侵犯您的权益,请联系客服反馈,我们会尽快为您处理(人工客服工作时间:9:00-18:30)。

问题:

吴老师你好,关于problems,在看过你下午传到邮箱里的纠正答案后,我仍有几个问题,希望老师能够帮助解答。

1. 关于子博弈精炼均衡和纳什均衡的区别,除了前者能剔除不可信威胁外,怎么从序贯理性的角度或者他们的定义解释二者差别呢?

Nash均衡主要主要是从静态角度来看问题的,所以它没有序贯理性的要求。即使对于动态博弈,Nash均衡实际上也是从静态的角度去分析的,因此,它会丢掉动态博弈中的一些重要信息,比如博弈的时序、博弈过程中的信息显示以及相应的信念更新,等等。而SPE强调的是博弈的动态性,所以有序贯理性的要求。

2. 第7题中问道:If (discount factor)is too small for the firms to use trigger strategies to sustain the monopoly output, what is the most-profitable symmetric subgame-perfect Nash equilibrium that can be sustained using trigger strategies?具体怎么理解呢?

要维持的合谋程度越高,要求的贴现因子越大。因为合谋程度越高,player背离的激励就越大,在惩罚手段一定的情况下(每一期博弈中,每个player i的minmax payoff是一定的,这个payoff就是能够队player i施加的最严厉的惩罚),要求的最小贴现因子就越大。

在贴现因子比较小,不足以维持完全合谋的情况下,只能维持部分合谋(比如在1至n1期合谋,n2至n3期实行Cournot竞争;然后再合谋n1期,------ )。这时候,参与人的payoff 就介于完全合谋和Cournot竞争之间。题目就是要求我们求出这个可以维持的最高的payoff。

3. 对于11题b博弈树,NE和SPNE是否是(R,M');两个博弈的PBE又如何求出呢?

对于这个博弈的NE,直接将其策略式表述写出来,用划线法就可以很容易得到。由于这个博弈只有原博弈这样一个子博弈,所以他的所有NE都是SPE。

其策略式表述为:

player 2

player 1 R 2, 4 2, 4 2, 4 L 1, 3 1, 2 4, 0 M 4, 0 0, 2 3, 3

该博弈的纯策略NE和SPE为(R,M')。

显然,很容易找到一个player 2的belief(比如p=1/2),使其同策略组合(R,M')一起构成一个WPBE。

实际上,对于任意的(13,23)

p ,player 2 都不会选择L'或R',这样,player 1的最优选择就是R。在这一策略组合下,player 2的信息集不在均衡路径上,所以p的值可以任意取。

另外,很容易验证,这个博弈没有混合策略纳什均衡。由于所有的WPBE中的策略组合都构成一个Nash均衡(纯策略或者混合策略),所以,这个博弈的WPBE必然包含策略组合(R,M')。

相关文档
最新文档